What set of numbers do -5 belong to?

Answers

Answer 1
The set of integers is a fundamental mathematical concept that includes all whole numbers (positive, negative, and zero) and their opposites.
The set of integers is denoted by the symbol Z and is a subset of the set of real numbers.
The set of integers is infinite in both directions, meaning that it extends infinitely both to the left and to the right on the number line.
Each integer is separated from its neighbors by a distance of 1 unit on the number line.
The set of integers is closed under addition, subtraction, and multiplication, which means that if you add, subtract or multiply any two integers, you will always get another integer.
However, division is not always closed under the set of integers, and may result in a fraction or decimal.
The integers are also closed under the operations of negation and absolute value.
The absolute value of an integer is always a non-negative integer.
The set of integers is countably infinite, which means that the integers can be listed in a sequence that includes every integer exactly once.
The set of integers is an important concept in many areas of mathematics, including algebra, number theory, and analysis.
The integers are used to represent quantities that can be counted, such as the number of apples in a basket or the number of students in a classroom.
Negative integers are used to represent quantities that have a direction opposite to that of positive integers.
The concept of integers is also used in computer science, where integers are used to represent data and perform calculations.
The set of integers is also used in many real-life situations, such as calculating distances between two points, measuring changes in temperature, or counting money.
The set of integers can be represented using set-builder notation, which uses braces to enclose the elements of the set and a vertical bar to separate the elements from the condition that defines them.
For example, the set of integers less than or equal to 5 can be represented as {…, -3, -2, -1, 0, 1, 2, 3, 4, 5}.
The set of integers can also be represented using interval notation, which uses brackets or parentheses to indicate whether the endpoints are included or excluded.
For example, the set of integers less than or equal to 5 can be represented as (-∞, 5].
The set of integers is an important concept for understanding more advanced mathematical concepts such as rational numbers, which are fractions that can be

Related Questions

Find the linear function

Answers

The linear function for this case is:

f(x) = 5,000*x + 7,000

How to find the linear function?

The general linear function is written as:

f(x) = a*x + b

Where a is the slope and b is the y-intercept.

Here we want a linear function for the given scenario, we know that the initial population is 7,000, then we can write:

f(x)= a*x + 7,000

Then we know that the population increases by 5,000 per year for 5 years, so the slope is 5,000, then we can write the function as:

f(x) = 5,000*x + 7,000

Learn more about linear functions at:

https://brainly.com/question/15602982

#SPJ1

Mr. Marshal spent his salary of $8 400 in the following manner: Rental .....1/5 Food.......1/10 Bank ......1/4 Miscellaneous ......... the remainder. what fraction of the money was spent on miscellaneous. how much did he spend on rental. if marshal spent 4/9 of miscellaneous on a trip what fraction of his entire salary was spent on the trip ?​

Answers

Mr. Marshal spent 1/4 of his salary on miscellaneous expenses and $1,680 on rental; therefore, he spent 1/36 of his entire salary on the trip.

To find the fraction of money spent on miscellaneous, we need to calculate the sum of the fractions spent on rental, food, and bank and subtract it from 1.

Rental: 1/5

Food: 1/10

Bank: 1/4

To find the fraction spent on miscellaneous:

Fraction spent on miscellaneous = 1 - (Rental + Food + Bank)

Rental + Food + Bank = 1/5 + 1/10 + 1/4 = (8 + 4 + 5)/40 = 17/40

Fraction spent on miscellaneous = 1 - 17/40 = 23/40

So, Mr. Marshal spent 23/40 of his salary on miscellaneous.

To find the amount spent on rental, we multiply the fraction spent on rental by his salary:

Amount spent on rental = (1/5) [tex]\times[/tex] $8,400 = $1,680

Therefore, Mr. Marshal spent $1,680 on rental.

If Mr. Marshal spent 4/9 of the miscellaneous amount on a trip, we need to calculate the fraction of his entire salary spent on the trip.

Fraction spent on the trip = (4/9) [tex]\times[/tex] (23/40) = 92/360 = 23/90

For similar question on fraction.

https://brainly.com/question/30154928  

#SPJ8

What is the volume of the triangular prism?
3 in.
15 in.
13 in.

Answers

Answer: 292.5 in^3

Explanation:
First use Pythagorean theorem to solve for the hypotenuse.

3^2 +15^2= c^2
15.3 =c
Next solve for the volume.

HELP ASAP!!!!!! LOOK AT THIS:
Alice, Raul, and Maria are baking cookies together. They need 3/4 cup of flour and 1/3 cup of butter to make a dozen cookies. They each brought the ingredients they had at home . Alice brought 2 cups of flour and 1/4 cup of butter, and Maria brought 1and1/4 cups of flour and 3/4 cup of butter. If the students have plenty of the other ingredients they need (sugar, salt, baking soda, etc.), how many whole batches of a dozen cookies can they make ?

Answers

,Alice, Raul, and Maria can make 1 whole batch of a dozen cookies together using the ingredients they brought.

To determine how many whole batches of a dozen cookies they can make, we need to compare the amount of flour and butter they have with the required amounts for each batch of cookies.

Let's calculate the total amount of flour and butter each student brought:

Alice:

Flour: 2 cups

Butter: 1/4 cup

Maria:

Flour: 1 1/4 cups

Butter: 3/4 cup

Now let's compare the amounts with the required ingredients for a batch of cookies:

Required amount per batch:

Flour: 3/4 cup

Butter: 1/3 cup

First, let's see how many batches of cookies Alice can make:

Flour: Alice has 2 cups, and each batch requires 3/4 cup. So she can make 2 cups / (3/4 cup) = 8/3 = 2 and 2/3 batches of cookies.

Butter: Alice has 1/4 cup, and each batch requires 1/3 cup. Since she doesn't have enough butter for a full batch, she can't make any batches of cookies with the butter she brought.

Next, let's see how many batches of cookies Maria can make:

Flour: Maria has 1 1/4 cups, and each batch requires 3/4 cup. So she canmake (5/4 cups) / (3/4 cup) = 5/3 = 1 and 2/3 batches of cookies.

Butter: Maria has 3/4 cup, and each batch requires 1/3 cup. So she can make (3/4 cup) / (1/3 cup) = 9/4 = 2 and 1/4 batches of cookies.

Now, to find the maximum number of whole batches they can make together, we take the minimum of the number of batches each student can make:

Alice: 2 and 2/3 batches

Maria: 1 and 2/3 batches

Raul: Not mentioned, so we assume he brought the required ingredients.

The minimum value is 1 and 2/3 batches, which means they can make 1 whole batch of a dozen cookies together.

For more such question dozen visit:

https://brainly.com/question/30948187

#SPJ8

The table below represents the function, and the following graph represents the function g.
4 -3 -2 -1 0 1
X-6 -5
f(x) 8-2
-8 -10 -8 -2 8 22
Complete the following statements.
The functions f and g have
The y-intercept of fis
the y-intercept of g.
Over the interval [-6, -31, the average rate of change of fis
m. All rights reserved.
9
-6 -4
-2
6
4
2
4-
N
6
2
4 6
the average rate of change of g

Answers

The correct options to the questions posed are :

The functions f and g have the same axis of symmetry.

The y-intercept of f is greater than the y-intercept of g.

Over the interval [-6, -3], the average rate of change of f is less than the average rate of change of g.

Axis of symmetry

From the given table of f(x) and x, from which we have;

The minimum point for f(x) as it's vertex as (-3, -10) = -3

For the function g(x). represented by the graph, the axis of symmetry is the vertical line passing through the vertex such that the y-values at equal distance from the line on either side are equal is the line x = -3

Intercept

The y-intercept, is the point on the graph where the line intersects the y-axis or where x = 0. Here , the point is (0,8) = 8

Over the interval [-6, -3]

The average rate of change of f(x) is

(-10 - 8)/(-3 -(-6)) = -6

Using the graph g(x)

From the graph of g(x), we have;

The axis of symmetry is the line x = -3

The y-intercept = (0, -2) = -2

Over the interval [-6, -3]

The average rate of change = (6 - (-2))/(-3 -(-6)) = 8/3 = 2.67

Hence,

The functions f and g have the same axis of symmetry.

The y-intercept of f is greater than the y-intercept of g.

Over the interval [-6, -3], the average rate of change of f is less than the average rate of change of g.

Learn more on functions : https://brainly.com/question/24320260

#SPJ1

answer a) and b) please

Answers

a : 10<x<=20

b: 20<x<=30

Find the equation of the line in slope-intercept form, perpendicular to a line with a slope of
1
13 and passing through (-1,3).
The equation of the line perpendicular to a line with a slope of -- and passing through (-1,3) is.
(Simplify your answer. Type your answer in slope-intercept form.)

Answers

The equation of the line perpendicular to the line with a slope of 1/13 and passing through the point (-1, 3) is y = -13x - 10.

To find the equation of a line perpendicular to a line with a slope of 1/13 and passing through the point (-1, 3), we need to determine the slope of the perpendicular line.

The slope of a line perpendicular to another line can be found by taking the negative reciprocal of the given slope. The negative reciprocal of 1/13 is -13/1, or simply -13.

Now that we have the slope (-13) and a point (-1, 3) that the line passes through, we can use the point-slope form of a linear equation to find the equation of the perpendicular line.

The point-slope form is given by:

y - y₁ = m(x - x₁)

where (x₁, y₁) is the given point and m is the slope.

Substituting the values (-1, 3) and -13 into the point-slope form, we get:

y - 3 = -13(x - (-1))

Simplifying:

y - 3 = -13(x + 1)

Expanding the equation:

y - 3 = -13x - 13

To convert the equation to slope-intercept form (y = mx + b), we can isolate y:

y = -13x - 13 + 3

y = -13x - 10

Therefore, the equation of the line perpendicular to the line with a slope of 1/13 and passing through the point (-1, 3) is y = -13x - 10.

for such more question on line perpendicular

https://brainly.com/question/18991632

#SPJ8

The surface area of a pyramid is the sum of the areas of the lateral faces and the area of the base.
O False
O True

Answers

Answer: True

Step-by-step explanation:

The lateral faces are the triangular faces that connect the apex of the pyramid to the edges of the base. The area of each lateral face can be calculated using the formula for the area of a triangle, which is [tex]\frac{1}{2} \times b \times h[/tex]. The area of the base is simply the area of the polygon that forms the base of the pyramid.

__________________________________________________________

SURFACE AREA

The surface area of a pyramid is the sum of the areas of the lateral faces and the area of the base. (True or False)

Answer:

The correct answer is True.

Explanation:

The surface area of a pyramid is the sum of the areas of the lateral faces and the area of the base. This can be represented by the formula:

[tex]\qquad\qquad\Large\boxed{\rm{\:SA = B + LA\:}}[/tex]

The lateral faces are the faces that are not the base, so their areas are calculated using the formula for the area of a triangle. The area of the base is calculated using the appropriate formula depending on the shape of the base.

Learn more about surface area here: https://brainly.com/question/32228774

Suppose a random sample of size 44 is selected from a population with =8 . Find the value of the standard error of the mean in each of the following cases (use the finite population correction factor if appropriate).

the size is n=500

the size is 5,000

the size is 50,000

Answers

Case 1: SE = s / √500

Case 2: SE = s / √5,000

Case 3: SE = s / √50,000

To find the standard error of the mean in each of the given cases, we can use the formula:

Standard Error (SE) = population standard deviation / square root of sample size

However, since the population standard deviation is not provided, we'll need to use an estimated value. We'll assume the population standard deviation is unknown and use the sample standard deviation as an estimate.

Given that the sample size is 44 and the population mean (μ) is 8, we'll calculate the standard deviation for each case:

Case 1: Sample size (n) = 500

The sample size (500) is larger than 5% of the population (44), so we can assume it's a large enough sample. In this case, we'll use the standard formula for the standard error of the mean without the finite population correction factor:

SE = sample standard deviation / square root of sample size

= s / √n

Case 2: Sample size (n) = 5,000

Similar to Case 1, the sample size (5,000) is larger than 5% of the population (44), so we'll use the standard formula without the finite population correction factor:

SE = s / √n

Case 3: Sample size (n) = 50,000

In this case, the sample size (50,000) is much larger than the population size (44), which means we have a large enough sample. Therefore, we'll also use the standard formula without the finite population correction factor:

SE = s / √n

for such more question on standard error

https://brainly.com/question/30404883

#SPJ8

what is question 4????

Answers

Answer:

2.2

Step-by-step explanation:

(1.85 x 2) - 1.50

...............

Research has shown that approximately 1 woman in 500 Carrie’s a mutation of particular gene. About 48% of women with this mutation develop colon cancer find the probability that a randomly selected woman will carry the mutation of this gene and will develop colon cancer

Answers

To find the probability that a randomly selected woman will carry the mutation of this gene and develop colon cancer, we need to multiply the probabilities of the two events: carrying the gene mutation and developing colon cancer.

Let's denote the probability of carrying the gene mutation as P(M) and the probability of developing colon cancer given the gene mutation as P(C|M). Given the information provided, P(M) can be calculated as 1 in 500, which is equivalent to 1/500.

The probability of developing colon cancer given the gene mutation is stated as 48%, which can be expressed as 0.48.

To find the probability of both events occurring, we multiply P(M) and P(C|M):

P(M and C) = P(M) * P(C|M) = (1/500) * 0.48

Therefore, the probability that a randomly selected woman will carry the mutation of this gene and develop colon cancer is (1/500) * 0.48, which can be calculated as a decimal or percentage based on your preference.

a. Find the open intervals on which the function is increasing and those on which it is decreasing.
b. Identify the​ function's local extreme​ values, if​ any, saying where they occur.

Answers

a. The open interval on which the function is increasing is [-2/3, 0].

The open intervals on which the function is decreasing are [-∞, -2/3] and [0, -∞].

b. The function's has a local maximum at (0, 0) and a local minimum at (-2/3, -4/27).

What is a decreasing function?

For any given function, y = f(x), if the output value (range or y-value) is decreasing when the input value (domain or x-value) is increased, then, the function is generally referred to as a decreasing function.

By critically observing the graph of the given function, we can reasonably infer and logically deduce that it is decreasing over the following open intervals:

increasing = [-2/3, 0].

decreasing = [-∞, -2/3] and [0, -∞].

Part b.

The local minimum of a graph is the point on the graph of a function where it changes from a decreasing function to an increasing function, which is given by this ordered pair (-2/3, -4/27);

h(x) = -x³ - x²

h'(x) = -3x² - 2x

0 = -3x² - 2x

3x² = -2x

3x = -2

x = -2/3

h(-2/3) = -(-2/3)³ - (-2/3)²

h(-2/3) = -4/27.

For the local maximum, we have:

(0, 0)

Read more on function here: brainly.com/question/9795474

#SPJ1

Choose the expression that is equivalent to fraction with 3 raised to the negative tenth power in the numerator and 3 raised to the fourth power times 3 raised to the zero power in the denominator.

Answers

Answer:

[tex]3^{-14}[/tex] or [tex]\displaystyle \frac{1}{3^{14}}[/tex]

Step-by-step explanation:

[tex]\displaystyle \frac{3^{-10}}{3^4*3^0}=\frac{3^{-10}}{3^4}=3^{-10-4}=3^{-14}[/tex]

PLSSSS HELPPP I WILL FIVE BRAINLY!!!!​

Answers

Answer:

The first box : -4  the second one : 6

Step-by-step explanation:

subtract 7 from 3 for the first box, then add 10 to -4 for the second one.

4) AD is a common internal tangent to circles B and C. Find the length of the radius
of circle B. Round to the nearest hundredth. (Hint: Prove that the two triangles
are similar and use proportions to find missing lengths.) (10 points)
I
B
E
6
D

Answers

Both triangles in the image are similar based on the AAA similarity theorem. The radius of the circle B is therefore calculated as: AB = 12.

What are similar triangles?

Similar triangles are geometric figures that have the same shape but may differ in size. They have corresponding angles that are equal and corresponding sides that are in proportion to each other.

Since AD serves as a common tangent, angle BAE is equal to 90 degrees, which is also equal to angle CDE due to being opposite angles.

By the Angle-Angle-Angle (AAA) similarity criterion, triangles ABE and DCE are similar.

Therefore:

AB/EA = DC/ED

Substitute:

AB/18 = 4/6

Cross multiply:

AB = 18 * 4/6

AB = 12

Therefore, the radius of the circle B is: 12.

Learn more about Similar triangles on:

https://brainly.com/question/27996834

#SPJ1

Which set of numbers are integers but not whole numbers or natural numbers?

Answers

Step-by-step explanation:  

C

A bakery is making cupcakes using a cylindrical mold. The cupcake mold has a diameter of 8.5 centimeters and is 6 centimeters tall. Which of the following shows a correct method to calculate the amount of cupcake batter needed to fill the mold all the way to the top? Use 3.14 for π.

V = (3.14)(8.5)2(6)
V = (3.14)(6)2(8.5)
V = (3.14)(4.25)2(6)
V = (3.14)(6)2(4.25)

Answers

Answer:

V = (3.14)(4.25)²(6)

Step-by-step explanation:

se logarithms to solve the problem.
The rule of 70 is a rule of thumb for estimating the doubling time of a quantity (e.g., investment, GDP, population) experiencing growth that is compounded continuously. The rule states that if the growth rate is r% per year, then the time it takes for the quantity to double is approximately 70/r years.

(a)
Use the rule of 70 to estimate the time it takes for an investment to double in value if it grows at the rate of 5% per year compounded continuously.
yr

(b)
What is the exact time it will take for the investment in part (a) to double in value? (Round your answer to two decimal places.)
yr

Answers

a.  The investment to double in value take about 14 years for the funding to double in value.

b.  The genuine time it will take for the funding to double in fee is about 13.86 years.

(a) To estimate the time it takes for an funding to double in cost the use of the rule of 70, we want to decide the increase rate. In this case, the increase price is given as 5% per 12 months compounded continuously.

Using the rule of 70, we can calculate the estimated doubling time:

Time to double ≈ 70 / boom rate

Time to double ≈ 70 / 5

Simplifying, we have:

Time to double ≈ 14 years

Therefore, it would take about 14 years for the funding to double in value.

(b) To decide the genuine time it will take for the funding to double in value, we can use the formulation for non-stop compounding:

Doubling time (exact) = ln(2) / (ln(1 + r))

where r is the increase fee as a decimal.

In this case, the increase charge is 5% per year, or 0.05 as a decimal.

Doubling time (exact) = ln(2) / (ln(1 + 0.05))

Doubling time (exact) ≈ 13.86 years (rounded to two decimal places)

Therefore, the genuine time it will take for the funding to double in fee is about 13.86 years.

For more related questions on investment:

https://brainly.com/question/34043637

#SPJ8

Results from a marijuana drug test study showed 143 subjects with positive test results including 24 false positive results. There were 157 negative​ results, including 3 false negative results. What is the probability that a randomly selected subject did not use​ marijuana? Round to three decimal places as needed.
Part 1
A. 0.533
B. 0.593
C. 0.523
D. 0.477

Answers

I think is c explanation it looked at me funny

Answer:

B. 0.593

Step-by-step explanation:

Nicole, Miguel, and Samuel served a total of 115 orders Monday at the school cafeteria. Miguel served 3 times as many orders as Samuel. Nicole served 10 more orders than Samuel. How many orders did they each serve?

Answers

Answer:

Samuel = 21 orders

Nicole = 31 orders

Miguel = 63 orders

Step-by-step explanation:

Let N represent Nicole's orders, M represents Miguel's orders, and S represent Samuel's orders.

We know that the sum of their tree orders equals 115 as

N + M + S = 115

Since Miguel served 3 times as many orders as Samuel, we know that

M = 3S.

Since Nicole served 10 more orders than Samuel, we know that

N = S + 10

Samuel's Orders:

Now we can plug in 3S for M and S + 10 for N to find S, the number of Samuel's orders:

S + 10 + 3S + S = 115

5S + 10 = 115

5S = 105

S = 21

Thus, Samuel served 21 orders.

Nicole's Orders:

Now we can plug in 21 for S in N = S + 10 to determine how many orders Nicole served:

N = 21 + 10

N = 31

Thus, Nicole served 31 orders.

Miguel's Orders:

Now we plug in 19 for S in M  = 3S to determine how many orders Miguel served:

M = 3(21)

M = 63

Thus, Miguel served 63 orders.

I only need help with the f(0)= the equation is above all the rest is filled in thank you ​

Answers

f(0) = -3

I believe, since the graph has a closed circle/point at (0,-3), f(0) should equal -3. Also, the graph 3x-3 has a domain of x>=0.

However, in terms of limits, the limit approaching x-->0 does not exist since the left and right limits do not equal one another.

Hope this helps.

The Hernandez family orders 3 large pizzas. They cut the pizzas so that each pizza has the same number of slices, giving them a total of 24 slices.
The Wilson family also orders several large pizzas from the same pizza restaurant. They also cut the pizzas so that all their pizzas have the same number of slices. For the Wilson family, the equation y = 10x represents the relationship, where x represents the number of pizzas and y represents the number of total slices.
Which statements best describe the pizzas bought by the Hernandez and Wilson families? Select two options.

Answers

The Hernandez family bought 3 large pizzas and cut them into equal slices.
The total number of slices they got was 24.
Therefore, each pizza was cut into 8 slices.
On the other hand, the Wilson family ordered several large pizzas.
They also cut the pizzas so that each pizza had the same number of slices.
The relationship between the number of pizzas and the total number of slices is given by y = 10x, where x is the number of pizzas and y is the total number of slices.
This means that each pizza was cut into 10 slices.
The Hernandez family got a total of 24 slices, while the Wilson family's total number of slices depends on the number of pizzas they ordered.
The pizzas bought by both families were large, and they both cut their pizzas into equal slices.

Type the correct answer in each box. Round your answers to the nearest thousandth.
A company has 200 machines. Each machine has 12% probability of not working.
If you were to pick 40 machines randomly, the probability that 5 would not be working is
and the probability that at least one machine would be working is
the probability that all would be working is

Answers

1) The probability that 5 will be working is: 0.187

2) The probability that at least one machine would be working is: 0.006

3) The probability that all would be working is : 1

How to find the probability of working?

We are given the parameters as:

Total number of machines = 200

Probability that a Machine is working = 12% = 0.12

1) Now, you want to pick 40 machines and want to find the probability that 5 will be working.

This probability is given by the expression:

P(5 working) = C(40,5) * 0.12⁵·0.88³⁵ ≈ 0.187

where C(n, k) = n!/(k!(n-k)!)

2) The probability that at least one machine would be working is:

0.88⁴⁰ ≈ 0.006

3) The probability that all would be working is the complement of the probability that all have failed. Thus:

P(all working) = 1 - 0.12⁴⁰ ≈ 1

Read more about probability of working at; https://brainly.com/question/25870256

#SPJ1

Select the correct answer from each drop-down menu.

Given: Line segment WU is the perpendicular bisector of line TV.

Prove: Line TU ≠ Line VU

Answers

WU is the perpendicular bisector of TV and W is the midpoint of TV.

Statements:

WU is the perpendicular bisector of TV.

W is the midpoint of TV.

TW = VW.

∠ZTWU and ∠ZVWU are right angles.

∠ZTWU ≅ ∠LVWU.

WU = WU.

∠ATWU ≅ ∠AVWU.

TU = VU.

Reasons:

Given.

Definition of the perpendicular bisector: It divides a segment into two congruent parts, and W is the midpoint of TV.

Definition of midpoint: The segment TW is congruent to VW because W is the midpoint of TV.

Given that WU is the perpendicular bisector, it means that ∠ZTWU and ∠ZVWU are right angles.

All right angles are congruent.

Reflexive property of congruence: Any segment or angle is congruent to itself.

Corresponding parts of congruent triangles are congruent (CPCTC): Since ∠ATWU and ∠AVWU are corresponding parts of congruent triangles, they are congruent.

Corresponding parts of congruent triangles are congruent (CPCTC): Since TU and VU are corresponding parts of congruent triangles, they are congruent.

Therefore, the correct statements and reasons are:

Statements:

WU is the perpendicular bisector of TV.

W is the midpoint of TV.

TW = VW.

∠ZTWU and ∠ZVWU are right angles.

∠ZTWU ≅ ∠LVWU.

WU = WU.

∠ATWU ≅ ∠AVWU.

TU = VU.

Reasons:

Given.

Definition of the perpendicular bisector.

Definition of midpoint.

Given.

All right angles are congruent.

Reflexive property of congruence.

CPCTC.

CPCTC.

Learn more about triangle here:

https://brainly.com/question/17335144

#SPJ8

Pablo used a total of 5 3/4 gallons of gas while driving his car. Each hour he was driving, he used 5/6 gallons of gas. What was the total number of hours he was driving? Write your ans

Answers

Let x be the number of hours Pablo was driving.

We know that he used 5/6 gallons of gas per hour of driving.

So, the total amount of gas he used is 5/6 * x.

We also know that he used a total of 5 3/4 gallons of gas.

So, we can set up the equation:

5/6 * x = 5 3/4

To solve for x, we can first convert 5 3/4 to an improper fraction:

5 3/4 = 23/4

Then, we can multiply both sides of the equation by the reciprocal of 5/6:

x = (5 3/4) / (5/6)

x = (23/4) / (5/6)

x = (23/4) * (6/5)

x = 27.6/4

x = 6.9

Therefore, Pablo was driving for a total of 6.9 hours.

brainliest???

0.059 and 0.01 which is greater?

Answers

0.059 is greater than 0.01

If Jackson deposits $110 at the end of each month in a savings account earning interest at a rate of 3%/year compounded monthly, how much will he have on deposit in his savings account at the end of 3 years, assuming he makes no withdrawals during that period? (Round your answer to the nearest cent.)

Answers

Answer:

The formula for calculating the future value (VF) of a periodic sum of money is:

VF = P * [(1 + r) n - 1] / r

where:

   VF is the future value (the total amount in the savings account)

   P is the periodic amount (monthly deposit)

   r is the periodic interest rate (annual interest rate divided by the number of periods in the year)

   n is the total number of periods (months)

In this case, P = $110, r = 3% / 12 = 0.03/ 12 = 0.0025 (monthly interest rate) and n = 3 * 12 = 36 (three years equivalent to 36 months).

Using these values in the formula, we can calculate the future value (VF):

VF = 110 * [(1 + 0.0025) 36 - 1] / 0.0025

Now let’s calculate this:

VF = 110 * [(1.0025) 36 - 1] / 0.0025

110 * (1.0965726572 - 1) / 0.0025

110 * 0.0965726572 / 0.0025

So Jackson will have about $4,239.52 in his savings account after three years, assuming he doesn’t make any withdrawals during that period.

Step-by-step explanation:

Study these equations:

f(x) = 2x – 4

g(x) = 3x + 1

What is h(x) = f(x)g(x)?

h(x) = 6x2 – 10x – 4
h(x) = 6x2 – 12x – 4
h(x) = 6x2 + 2x – 4
h(x) = 6x2 + 14x + 4

Answers

Answer:

6x2-10x-4

Step-by-step explanation:

hx=(2x-4)(3x+1)

hx=2x(3x+1)-4(3x+1)

hx=6x2+2x-12x-4

hx=6x2-10x-4

please help!! need it fast, will give brainliest!! and pls show work !!

Find the measure of angle AEB

Answers

Answer:

An acute angle

Step-by-step explanation:

An acute angle is smaller than an obtuse ad right angle.

hope this helps and hope it was right 'cause I really don't know what you meant. :)

In 1995, wolves were introduced into Yellowstone Park.



The function `w\left(x\right)=14\cdot1.08^{x}` models the number of wolves, `w`, in the years since 1995, `x`.



By what percent does the number of wolves change each year?

Answers

In this manner, the number of wolves changes by around 8 percentage 8% each year based on the given work.

Percent calculation.

To determine the percentage change within the number of wolves each year, we ought to look at the development rate of the work w(x) = 14 * 1.08^x.

The development rate in this case is given by the example of 1.08, which speaks to the figure by which the number of wolves increments each year. In this work, the coefficient 1.08 speaks to a development rate of 8% per year.

To calculate the percentage change, we subtract 1 from the growth rate and increase by 100 to change over it to a rate:

Percentage change = (1.08 - 1) * 100 = 0.08 * 100 = 8%.

In this manner, the number of wolves changes by around 8 percentage 8% each year based on the given work.

Learn more about percentage below.

https://brainly.com/question/24304697

#SPJ4

Other Questions
from smallest to largest, what is the correct order of the following distances? (ly stands for light year) 1 ly, 1 km, 1 AU, 1 cm you are studying the effects of climate change on the geographic ranges of species living in the mountains, and your climate records show that mean temperatures are increasing. from a conservation standpoint, which species would you be most concerned about? HeyCan you help me out on this? I also need a sketchUse the following information to answer the next question The function y=f(x) is shown below. 20. Describe the transformation that change the graph of y=f(x) to y=-2 f(x+4)+2 and ske The Fibonacci numbers {fi} are defined recurrently by f1=1f2=1f3=f1+f2fn=fn1+fn2 Use Euclidean lemma to prove that gcd(fn,fn+1)=1 for every nN. The Language Arts department conducts a study to see if the number of books a student reads per month affects the score on the SAT Verbal Test. Here is the data that the Language Arts department collected for 8 students. Create the scatter plot for this data set. What is the equation of the line of best fit? Let U,V,W be finite dimensional vector spaces over F. Let SL(U,V) and TL(V,W). Prove that rank(TS)min{rank(T),rank(S)}. 3. Let V be a vector space, TL(V,V) such that TT=T. Now you are ready to implement parse_arguments (). If you find a name, you have to access the argument after it. A for loop makes this awkward: a whi le loop is easier. Begin with this code: index =1 while index < len (sys.argv): arg = sys. argv[index] index +=1 arg is a name, you should - figure out if the name is "width" or "height". - increment index, and retrieve the next argument (which is a value). - remember to convert the value into an int! - change either the width or height variable. arg is not a name, then it's a positional argument. In this case, you should just store it into symbol. the primary goal of external monitoring is to maintain an informed awareness of the state of all of the organizations networks, information systems, and information security defenses.true or false scholar robert putnam argues that the more ________________ a community has, the greater the level of trust among its members, and the easier it is to achieve common goals outside of government. research studies have shown that the risk for developing type 2 diabetes and hypertension begins to increase at a bmi of greater than: a. 24 kg/m2. b. 30 kg/m2. c. 35 kg/m2. d. 40 kg/m2. Find the linearization of f(x, y, z) = x/,yzat the point (3, 2, 8).(Express numbers in exact form. Use symbolic notation and fractions where needed.) when meis family moved to the united states, she tried to dress and talk like her american classmates. in terms of relating to the dominant group, what goal was she attempting to accomplish? Uncertain Future Cash Flows Lukow Products is investigating the purchase of a piece of automated equipment that will save $400,000 each year in direct labor and inventory carrying costs. This equipment costs $2,500,000 and is expected to have a 15-year useful life with no salvage value. The companys required rate of return is 20% on all equipment purchases. Management anticipates that this equipment will provide intangible benefits such as greater flexibility and higher-quality output that will result in additional future cash inflows. Required: 1. What is the net present value of the piece of equipment before considering its intangible benefits? 2. What minimum dollar value per year must be provided by the equipments intangible benefits to justify the $2,500,000 investment? Plants commonly develop in unsuitable environments and areusually subjected to various stresses during development. Discussheat stress, ultraviolet stress MTI makes three types of lawn tractors: M3100, M4100, and M6100. In the past, it allocated overhead to products using machine-hours. Last year, the company produced 9,000 units of M3100, 16,000 units of M4100, and 8,000 units of M6100 and had the following revenues and costs: MTI Income Statement M3100 M4100 M6100 Total $ 9,100,000 $ 15,900,000 $ 13,900,000 $ 38,900,000 2,500,000 570,000 4,200,000 890,000 3,500,000 1,790,000 10,200,000 3,250,000 Sales revenue Direct costs Direct materials Direct labor Variable overhead Setting up machines Processing sales orders Warehousing Operating machines Shipping Contribution margin Plant administration Gross profit 2,304,000 1,936,000 2,604,000 1,250,000 858,000 $ 16,498,000 6,000,000 $ 10,498,000 MTI's controller has heard about activity-based costing and puts together an employee team to recommend cost allocation bases. The employee team recommends the following: Calculate MIPS:frequency: 200 MHz, so I think clockrate is 1/200 which is 0.005CPI: 4.53total instruction count: 15apparently the answer is 44.12 but I have no idea how to get that number. Maybe I am calculating it wrong? I used the formula: clockrate / CPI / 10^6.Please let me know how to calculate MIPS or if you think you know what I am doing wrong in representing the physical features of the eagle, the transformation mask supports which of the following? is journalism an effective tool to reform american politics and society? a) What is the purpose of regularization? b) State the loss functions of linear regression and logistic regression under regularization (choose any regularization method you like). which of the following objects demonstrates use of composite view technique?